LSAT and Law School Admissions Forum

Get expert LSAT preparation and law school admissions advice from PowerScore Test Preparation.

 Administrator
PowerScore Staff
  • PowerScore Staff
  • Posts: 8916
  • Joined: Feb 02, 2011
|
#72663
Complete Question Explanation

Resolve the Paradox. The correct answer choice is (E).

Answer choice (A):

Answer choice (B):

Answer choice (C):

Answer choice (D):

Answer choice (E): This is the correct answer choice.


This explanation is still in progress. Please post any questions below!
User avatar
 JocelynL
  • Posts: 51
  • Joined: Dec 22, 2020
|
#83664
Hello,
I got this question correct, but was still tempted towards a few of the other answer choices. Can someone please let me know if my reasoning is accurate in why these are wrong:

We are looking for an a/c that explains why although wild animals have very high levels of radiation, their population has expanded rapidly.

A- this was a tempting a/c. In a sense it does explain both sides, by stating that animals arrived in the area after the accident so the animals are migrating to the area. But is it safe to say that just because they are migrating to the area doesn't really mean its expanding rapidly. It could just be a few animals. Also, does it matter that the a/c talks about "animals" and not "wildlife" specifically?

B/ C/ D - None of these explain how the wild life population has expanded rapidly

E - As soon as I read this one, I realized it was a much better a/c than A so I chose it.
User avatar
 KelseyWoods
PowerScore Staff
  • PowerScore Staff
  • Posts: 1079
  • Joined: Jun 26, 2013
|
#83728
Hi Jocelyn,

Your reasoning is accurate! Answer choice (A) doesn't really explain why the wildlife in the area would be rapidly expanding because there could just be a few animals migrating in and it seems like the animals migrating in are also developing high levels of radiation so it's not like they are any better off than the animals that were there before the accident. The wildlife vs. animals thing might also be an issue here, but the bigger problem is just that it doesn't even explain why there would be more animals.

And you're exactly right about B/C/D as well--they do not explain why the wildlife in the area rapidly expanded even though there's high radiation levels.

Answer choice (E) is the only one which explains the expansion in wildlife and, thus, it is the most attractive (and only correct!) answer choice.

Hope this helps!

Best,
Kelsey

Get the most out of your LSAT Prep Plus subscription.

Analyze and track your performance with our Testing and Analytics Package.